M/A 1 is easy to guess: if f5 is possible then f6 is also possible, so by the conventions of retrograde analysis the answer must be f6. Proving that the pawn cannot be on f5 is harder, and I almost wrote to you that the position is impossible. The White bishop cannot have started at f1, and must therefore be the result of an underpromotion. Since there had to be a path to the corner, the pawn on b3 must have started at b2 and advanced after the bishop moved to the corner. It follows that the other advanced White pawns started on d2 & f2, so the underpromoted pawn started at a2. Since the exits from a8 & c8 are blocked, the promotion must have happened at e8 or g8. To bypass Black's KP, the pawn must have made 6 captures. These cannot include Black's QB (which obviously never moved) or the KB (which stayed on black squares) or Black's KKtP (which could at most move diagonally toward a1). That leaves 7 pieces that the White pawn could capture: a) The Black queen; b) Both Black knights; c) Black's KBP, or whatever it was promoted to; d) A Black rook; e) Whatever the Black KKtP was promoted to; f) Another Black rook, but only if a Black pawn was promoted to a rook and moved to a8. Thus the KKtP must have been promoted. (Promoting the KBP doesn't increase the number of captures available to the White pawn.) Passing through d2 or f2 would force the White king to move, so it must have reached b2, making 5 captures. Since White's KB never moved, the Black pawn must have captured all the other missing White pieces. Moreover, White's QKtP had to move to b3 before the Black pawn could be promoted, so the White bishop was already in the corner, and we can rule out (e). Thus Black promoted a pawn to a rook on b1, but no White piece was available to interpose. (The QR has not moved, and capturing the new Black rook would not allow it to reach a8.) Thus the check was blocked by a Black piece, which can only be the KB. Later the KB was captured by a White pawn, but there is only one pawn capture unaccounted for. Thus the pawn on the KB file must have advanced without capturing, and the pawn on e4 must have started at d2 and captured the bishop on e3. Finally, we come to the mystery pawn. Since it hasn't captured anything, it must have advanced at least to f4 before White's KR could throw itself in front of Black's KKtP. After b1=R by Black, most White pieces were already in the positions shown. The bishop could not move except to capture the Black rook, and the QP had to wait on d2 to capture the Black bishop. White's only move is thus to push the KBP. The Black pieces had to leave the corner, but a bishop move would have discovered check and there was still a pawn on d2, so there must have been a sequence like 1. ... Rb2 2. Bb1 Ra2 (The Black king & White bishop could dance when the rook is on b2, but the result is the same.) Once again, White's only move is a push, so the pawn must be on f6. As shown below, the position is possible. 1. Nc3 Nf6 2. Nd5 Ne4 3. Nb6 Ng3 4. Nxc8 Nxf1 5. Nb6 Ng3 6. Nd5 Ne4 7. Nf6+ gxf6 8. Nf3 Bh6 9. Ne5 Be3 10. Rf1 Bc5 11. f4 Bb4 12. Rf3 Nc5 13. Rd3 Nb3 14. axb3 Nc6 15. Rd4 Na5 16. Re4 Nc4 17. bxc4 Kf8 18. Re3 Kg7 19. Rg3+ Kh6 20. Rd3 Rg8 21. Rd4 fxe5 22. Re4 Rg6 23. Re3 Rd6 24. Re4 Rd5 25. cxd5 Qh8 26. Re3 Qf6 27. Rd3 Qe6 28. dxe6 a6 29. dxf7 Rg8 30. fxg8=B c6 31. Ba2 Kg7 32. b3 Bc5 33. Bb2 Ba3 34. Bd4 exd4 35. Rc3 dxc3 36. Qc1 Bb4 37. Qa3 Bc5 38. Qa5 Ba3 39. Bb1 Bc1 40. Qa3 Kg8 41. Qb2 cxb2 42. Ba2 b1=R 43. f5 Rb2 44. Bb1 Ra2 45. f6 Ra5 46. Ba2 Rg5 47. Bb1 Kh8 48. Ba2 Rg8 49. Bb1 Bb2 50. Ba2 Bd4 51. Bb1 Be3 52. dxe3 Ra8 53. Ba2 Kg8 54. Bb1 Kf8 55. e4 Now 55. ... Ke8 56. Ba2 gives the position in the problem with Black to move. 55. ... Kf7 56. Ba2 Ke8 gives the same position with White to move.